Đến nội dung

Hình ảnh

[TOPIC] Nhiều cách giải cho một bài toán. Bạn chọn cách nào?

* * * * * 4 Bình chọn

  • Please log in to reply
Chủ đề này có 17 trả lời

#1
Crystal

Crystal

    ANGRY BIRDS

  • Hiệp sỹ
  • 5534 Bài viết
Topic này sẽ giới thiệu đến các bạn một số bài toán và những cách giải hay, sáng tạo cho những bài toán đó (ít nhất cũng được 2 cách)

Phạm vi hoạt động: THCS, THPT (đặt ở THPT)

Các loại toán: Bất đẳng thức - Cực trị; Phương trình - Hệ phương trình - Bất phương trình; ....

Đang tổng hợp ...

Rất cần sự trợ giúp của các MOD.

#2
Tham Lang

Tham Lang

    Thượng úy

  • Thành viên
  • 1149 Bài viết
MOD mới xin được thỉnh giáo :D
Một bài toán cũ, mình đã post ở Olympic, và nhận được một lời giải khá hay, nhưng bây giờ, mình sẽ post thêm vài ba cách nữa. Thực ra, những cách này chỉ để chỉ rõ sự yếu ớt của BĐT này, và để cho mọi người vùi nó cho ngon lành :))
Bài toán :
Cho $a,b,c$ là các số thực dương thoả mãn $a+b+c=3$. Chứng minh rằng :
$$(ab+bc+ca)\left (\dfrac{a}{b^2+9}+\dfrac{b}{c^2+9}+\dfrac{c}{a^2+9}\right )\le \dfrac{9}{10}$$
  • Cách 1. [ Võ Quốc Bá Cẩn]
Bất đẳng thức tương đương :
$$\sum \left (\dfrac{a}{b^2+9}\right )\le \dfrac{9}{10(ab+bc+ca)}$$
$$\Leftrightarrow \dfrac{9}{10(ab+bc+ca)}+\sum \dfrac{\dfrac{1}{9}ab^2}{b^2+9}\ge \dfrac{1}{3}$$
Sử dụng BĐT CS, ta có :
$$\sum \dfrac{ab^2}{b^2+9}\ge \dfrac{(ab+bc+ca)^2}{ab^2+bc^2+ca^2+27}$$
Ta cần chứng minh
$$\dfrac{9}{10(ab+bc+ca)}+\dfrac{(ab+bc+ca)^2}{ab^2+bc^2+ca^2+27}\ge \dfrac{1}{3}$$
Sử dụng $AM-GM$, ta có :
$$\dfrac{2}{5(ab+bc+ca)}+\dfrac{(ab+bc+ca)^2}{9(ab^2+bc^2+ca^2+27)}\ge 5\sqrt[5]{\dfrac{1}{9.10^4.(ab+bc+ca)^4(ab^2+bc^2+ca^2+27)}}$$
Lại sử dụng AM-GM, ta có :$$(ab+bc+ca)^4(ab^2+bc^2+ca^2)\le (ab+bc+ca)^4(a^2b^2+b^2c^2+c^2a^2)(a^2+b^2+c^2)$$
$$\le 27(ab+bc+ca)^2(a^2b^2+b^2c^2+c^2a^2)$$
Đặt $ab+bc+ca=x$. Theo BĐT AM-GM, và BĐT Schur, ta có :
$$(ab+bc+ca)^2(a^2b^2+b^2c^2+c^2a^2)=x^2(x^2-6abc)\ge x^2(x^2-18x+18)=27+(x-3)^2(x+1)\ge 27$$
Do đó $(ab+bc+ca)^2(ab^2+bc^2+ca^2)\le 27$
Suy ra
$$\dfrac{2}{5(ab+bc+ca)}+\dfrac{(ab+bc+ca)^2}{9(ab^2+bc^2+ca^2+27)}\ge \dfrac{1}{6}$$
$$\Leftrightarrow \dfrac{9}{10(ab+bc+ca)}+\dfrac{(ab+bc+ca)^2}{9(ab^2+bc^2+ca^2)}\ge \dfrac{1}{3} \Box$$
  • Cách giải 2. [Tham Lang]
Áp dụng BĐT AM-GM, ta có :
$$(ab+bc+ca)\left (\dfrac{a}{b^2+9}+\dfrac{b}{c^2+9}+\dfrac{c}{a^2+9}\right )\le \dfrac{1}{2}(ab+bc+ca)\left (\dfrac{a}{b+4}+\dfrac{b}{c+4}+\dfrac{c}{a+4}\right )$$
Ta cần chứng minh :
$$(ab+bc+ca)\left (\dfrac{a}{b+4}+\dfrac{b}{c+4}+\dfrac{c}{a+4}\right )\le \dfrac{9}{5}$$
$$\Leftrightarrow \dfrac{(ab+bc+ca)\left [ab^2+bc^2+ca^2+84-4(ab+bc+ca)\right ]}{abc+4(ab+bc+ca)+112}\le \dfrac{9}{5}$$
Ta lại có :
$$\dfrac{ab^2+bc^2+ca^2+84-4(ab+bc+ca)}{abc+4(ab+bc+ca)+112}+1=\dfrac{ab^2+bc^2+ca^2+abc+196}{abc+4(ab+bc+ca)+112}$$
Sử dụng BĐT phụ $ab^2+bc^2+ca^2+abc\le 4$
Thật vậy, giả sử $b$ nằm giữa $a,c$ suy ra $a(b-a)(b-c)\le 0\Leftrightarrow ab^2+a^2c\le abc+a^2b\Leftrightarrow ab^2+bc^2+ca^2+abc\le a^2b+bc^2+2abc=b(a+c)^2=\dfrac{2b(a+c)^2}{2}\le \dfrac{4(a+b+c)^3}{27}=4$
Do đó :
$$\dfrac{(ab+bc+ca)(ab^2+bc^2+ca^2+84-4(ab+bc+ca)}{abc+4(ab+bc+ca)+112}\le (ab+bc+ca)\left [\dfrac{200}{abc+4(ab+bc+ca)+112}-1\right ]$$
Đặt $ab+bc+ca=q, abc=r$. Ta sẽ chứng minh :
$$q\left (\dfrac{200}{r+4q+112}-1\right )\le \dfrac{9}{5}$$
$$\Leftrightarrow 404q\le 5qr+20q^2+9r+1008$$
Giờ hơi mệt, tí nữa gõ tiếp ... :wacko: (tí nữa sẽ có thể phần thành 2 cách giải hoàn toàn riêng biệt )...

Đến đây
  • Cách 2.1
Áp dụng BĐT Schur, ta có $r\ge \dfrac{p(4q-p^2)}{9}$
Lúc này
$$20q^2+5qr+9r-404q+1008\ge 20q^2+\dfrac{5q(q-9)}{3}+3(4q-9)-404q+1008\ge 0$$
$$\Leftrightarrow (80q-981)(q-3)\ge 0$$
Hiển nhiên đúng vì $q\le 3$
  • Cách 2.2
Nhắc lại bổ đề :
Nếu $a,b,c$ là các số thực không âm thì $a^2+b^2+c^2+2abc+1\ge 2(ab+bc+ca)$ Việc chứng minh BĐT này đã được thực hiện rất nhiều nên mình xin phép không chứng minh lại.
Áp dụng bổ đề, ta có :
$$5qr+75=5(ab+bc+ca)abc+5\dfrac{(a+b+c)(a+b+c)^2}{2}+\dfrac{15}{2}$$
$$\ge \dfrac{5(ab+bc+ca).2abc+5(ab+bc+ca)\left [a^2+b^2+c^2+2(ab+bc+ca)\right ]+5(ab+bc+ca)}{2}$$
$$=\dfrac{5(ab+bc+ca)(2abc+a^2+b^2+c^2+1)}{2}+5(ab+bc+ca)^2$$
$$\ge 5(ab+bc+ca)^2+5(ab+bc+ca)^2=10q^2$$
$$5qr+75\ge 10q^2\Leftrightarrow 5qr+165\ge 10q^2+90\ge 60q$$
Tiếp
$$9r+45=9abc+\dfrac{9(a+b+c)^2}{2}=\dfrac{9(2abc+a^2+b^2+c^2+1}{2}+9(ab+bc+ca)\ge 18q$$

$$20(q^2+9)\ge 120q$$
Nên ta suy ra
$$5qr+20q^2+9r+390\ge 198q$$
Vậy, chỉ còn cần chứng minh $206q\le 618 \Leftrightarrow q\le 3$ Hiển nhiên đúng.
Từ các cách giải trên đây, có thể cho thấy, BĐT này vô cùng lỏng, và mình mong đợi sẽ có thêm được nhiều cách giải hay hơn từ chính các bạn :D

Off vĩnh viễn ! Không ngày trở lại.......


#3
Crystal

Crystal

    ANGRY BIRDS

  • Hiệp sỹ
  • 5534 Bài viết
Cảm ơn Tham Lang đã tham gia cùng anh. Nhưng với một bài toán thì em nên gửi ngay trong một post thôi à. Như thế sẽ dễ theo dõi hơn.

Một lần nữa cảm ơn em đã cùng anh song tấu :D

#4
LilTee

LilTee

    Binh nhất

  • Thành viên
  • 26 Bài viết
Sử dụng bất đẳng thức $AM-GM$, ta có : $$(ab+bc+ca)\left (\dfrac{a}{b^2+9}+\dfrac{b}{c^2+9}+\dfrac{c}{a^2+9}\right )\le \dfrac{1}{2}(ab+bc+ca)\left (\dfrac{a}{b+4}+\dfrac{b}{c+4}+\dfrac{c}{a+4}\right )$$ Do đó bài toán sẽ được chứng minh nếu ta chỉ ra được
$$(ab+bc+ca)\left (\dfrac{a}{b+4}+\dfrac{b}{c+4}+\dfrac{c}{a+4}\right )\le \dfrac{9}{5}$$ hay tương đương
$$\dfrac{(ab+bc+ca)\left [ab^2+bc^2+ca^2+84-4(ab+bc+ca)\right ]}{abc+4(ab+bc+ca)+112}\le \dfrac{9}{5}$$ Sử dụng bổ đề quen thuộc $$(ab+bc+ca)(ab^2+bc^2+ca^2) \le 9,$$ ta đưa bài toán về chứng minh $$9\left( r+4q+112 \right)\ge 5\left( 9+84q-4{{q}^{2}} \right)$$ hay tương đương $$9r+20{{q}^{2}}-384q+963\ge 0.$$ Theo $Schur$, ta có $9r \ge 3(4q-9)$, do đó $$\begin{align}
9r+20{{q}^{2}}-384q+963 & \ge 3\left( 4q-9 \right)+20{{q}^{2}}-384q+963 \\
& =4\left( q-3 \right)\left( 5q-78 \right) \\
& \ge 0. \\
\end{align}$$ Có được đánh giá cuối vì $q\le3$. Bài toán được chứng minh xong ;).
Đẳng thức xảy ra khi và chỉ khi $a=b=c=1. \ \blacksquare$

Diễn đàn Vật lí phổ thông: https://vatliphothong.vn
My Blog: http://tanghaituan.com

Học trực tuyến: https://hoctructuyen.tv


#5
Tham Lang

Tham Lang

    Thượng úy

  • Thành viên
  • 1149 Bài viết
Tiếp tục là một bài toán khá hay, mà theo mình sẽ có rất nhiều cách giải. Mình post đề trước đã, mọi người thử làm rồi mình sẽ post lời giải của mình sau :
Bài toán
Cho $a,b,c$ là các số thực không âm thoả mãn $ab+bc+ca \ne 0$. Chứng minh rằng :
$$\left (ab+\dfrac{c}{a+b}\right )\left (bc+\dfrac{a}{b+c}\right )\left (ca+\dfrac{b}{c+a}\right )\le \dfrac{1}{4}$$

Tập huấn đội tuyển 2009 -Phạm Kim Hùng


Bài viết đã được chỉnh sửa nội dung bởi Tham Lang: 11-07-2012 - 02:44
Khác!!!

Off vĩnh viễn ! Không ngày trở lại.......


#6
Crystal

Crystal

    ANGRY BIRDS

  • Hiệp sỹ
  • 5534 Bài viết
BÀI TOÁN. Giải phương trình: $\mathbf{{x^2} - 2x - 3 = \sqrt {x + 3}} $

Yêu cầu: Giải bằng 10 cách nhé.

$\boxed{\textbf{Lời giải 1 - minhdat881439}}$

Điều kiện: $x\geq -3$. Phương trình đã cho tương đương với:
\[{x^2} - x - \frac{1}{4} = (x + 3) + \sqrt {x + 3} + \frac{1}{4} \Leftrightarrow {(x - \frac{1}{2})^2} = {(\sqrt {x + 3} + \frac{1}{2})^2}\]
\[ \Leftrightarrow \left[ \begin{array}{l}
x - \frac{1}{2} = (\sqrt {x + 3} + \frac{1}{2})\\
x - \frac{1}{2} = - (\sqrt {x + 3} + \frac{1}{2})
\end{array} \right. \Leftrightarrow \left[ \begin{array}{l}
x - 1 = \sqrt {x + 3} \\
- x = \sqrt {x + 3}
\end{array} \right. \Leftrightarrow \left[ \begin{array}{l}
x = \frac{{3 + \sqrt {17} }}{2}\\
x = \frac{{1 - \sqrt {13} }}{2}
\end{array} \right.\]
Vậy phương trình đã cho có nghiệm là $x = \left\{ {\frac{{1 - \sqrt {13} }}{2};\frac{{3 + \sqrt {17} }}{2}} \right\}$

$\boxed{\textbf{Lời giải 2 - nthoangcute}}$

Điều kiện: $x\geq -3$.
\[{x^2} - 2x - 3 = \sqrt {x + 3} \Rightarrow {({x^2} - 2x - 3)^2} = x + 3 \Leftrightarrow ({x^2} - x - 3)({x^2} - 3x - 2) = 0\]
\[ \Leftrightarrow \left[ \begin{array}{l}
{x^2} - x - 3 = 0\\
{x^2} - 3x - 2 = 0
\end{array} \right. \Leftrightarrow \left[ \begin{array}{l}
x = \frac{{3 \pm \sqrt {17} }}{2}\\
x = \frac{{1 - \sqrt {13} }}{2}
\end{array} \right.\]
Thử lại thấy $x=\frac{1-\sqrt{17}}{2}$ và $x=\frac{3+\sqrt{17}}{2}$ thỏa mãn đề bài.

Vậy $x=\frac{1-\sqrt{17}}{2}$ hoặc $x=\frac{3+\sqrt{17}}{2}$

$\boxed{\textbf{Lời giải 3 - nthoangcute}}$

Điều kiện: $x\geq -3$. Đặt $\sqrt {x + 3}=y \geq 0$. Suy ra $x=y^2-3$

Vậy phương trình đã cho tương đương với:
\[{({y^2} - 3)^2} - 2({y^2} - 3) - 3 = y \Leftrightarrow ({y^2} - y - 4)({y^2} + y - 3) = 0\]
\[ \Leftrightarrow \left[ \begin{array}{l}
y = \frac{{ - 1 + \sqrt {13} }}{2}\\
y = \frac{{1 + \sqrt {17} }}{2}
\end{array} \right.\,\,\,\,\left( {y \ge 0} \right) \Rightarrow \left[ \begin{array}{l}
x = \frac{{1 - \sqrt {17} }}{2}\\
x = \frac{{3 + \sqrt {17} }}{2}
\end{array} \right.\]
Vậy ta được $x=\frac{1-\sqrt{17}}{2}$ hoặc $x=\frac{3+\sqrt{17}}{2}$

$\boxed{\textbf{Lời giải 4 - luxubuhl}}$

Điều kiện: $x\geq -3$. Đặt $\sqrt{x+3}=t-1$

\[ \Rightarrow \left\{ \begin{array}{l}
{t^2} - 2t + 1 = x + 3\\
{x^2} - 2x - 3 = t - 1
\end{array} \right. \Leftrightarrow \left\{ \begin{array}{l}
{t^2} - 2t - x - 2 = 0\\
{x^2} - 2x - t - 2 = 0
\end{array} \right.\]
\[ \Rightarrow (t - x)(t + x) - 2(t - x) + (t - x) = 0 \Rightarrow (t - x)(t + x - 1) = 0\]
\[ \Rightarrow \left[ \begin{array}{l}
t = x\\
t = 1 - x
\end{array} \right. \Leftrightarrow \left[ \begin{array}{l}
\sqrt {x + 3} + 1 = x\\
\sqrt {x + 3} = x
\end{array} \right. \Leftrightarrow \left[ \begin{array}{l}
x = \frac{{3 + \sqrt {17} }}{2} - True\\
x = \frac{{1 - \sqrt {13} }}{2} - True
\end{array} \right.\]

$\boxed{\textbf{Lời giải 5 - nthoangcute}}$

Điều kiện: $x\geq -3$.
\[{x^2} - 2x - 3 = \sqrt {x + 3} \Leftrightarrow (x + \sqrt {x + 3} )(x - 1 - \sqrt {x + 3} ) = 0\]
Đây là phương trình tích nên dễ dàng giải được: $x=\frac{1-\sqrt{17}}{2}$ hoặc $x=\frac{3+\sqrt{17}}{2}$

$\boxed{\textbf{Lời giải 6 - donghaidhtt}}$

Điều kiện: $x\geq -3$.
\[{x^2} - 2x - 3 = \sqrt {x + 3} \Leftrightarrow {x^2} - 3x - 2 = \sqrt {x + 3} - (x - 1)\]

\[ \Leftrightarrow \left[ \begin{array}{l}
\sqrt {x + 3} = - (x - 1)\\
\left\{ \begin{array}{l}
\sqrt {x + 3} \ne - (x - 1)\\
{x^2} - 3x - 2 = \frac{{ - {x^2} + 3x + 2}}{{\sqrt {x + 3} + (x - 1)}}
\end{array} \right.
\end{array} \right.\]

$\boxed{\textbf{Lời giải 7 - donghaidhtt}}$

Điều kiện: $x\geq -3$.

\[{x^2} - 2x - 3 = \sqrt {x + 3} \Leftrightarrow {x^2} - x - 3 = \sqrt {x + 3} + x \Leftrightarrow \left[ \begin{array}{l}
\sqrt {x + 3} = x\\
\left\{ \begin{array}{l}
\sqrt {x + 3} \ne x\\
{x^2} - x - 3 = \frac{{ - {x^2} + x + 3}}{{\sqrt {x + 3} - x}}
\end{array} \right.
\end{array} \right.\]
* 2 cách này tương tự nhau nên làm thử với cách 7:

\[\left[ \begin{array}{l}
\sqrt {x + 3} = x\\
\left\{ \begin{array}{l}
\sqrt {x + 3} \ne x\\
{x^2} - x - 3 = \frac{{ - {x^2} + x + 3}}{{\sqrt {x + 3} - x}}
\end{array} \right.
\end{array} \right. \Leftrightarrow \left[ \begin{array}{l}
\left\{ {\begin{array}{*{20}{c}}
{x \ge 0}\\
{{x^2} - x - 3 = 0}
\end{array}} \right.\\
\left\{ {\begin{array}{*{20}{c}}
{{x^2} - x - 3 \ne 0}\\
{ - 1 = \sqrt {x + 3} - x}
\end{array}} \right.
\end{array} \right.....\]

#7
Crystal

Crystal

    ANGRY BIRDS

  • Hiệp sỹ
  • 5534 Bài viết
BÀI TOÁN. Giải hệ phương trinh: $\left\{\begin{array}{l} {x^2} - 4y + 4 = 0\\ {y^2} - 4x + 4 = 0 \end{array} \right.$

>> XEM

$\boxed{\textbf{Lời giải 1 - davildark}}$

Cộng 2 vế $(1)$ và $(2)$ ta có $ x^2-4x+4+y^2-4y+4=0$
$$\Leftrightarrow (x-2)^2+(y-2)^2=0\Rightarrow x=2=y$$
Vậy nghiệm của hệ phương trình là $\boxed{\text {x=y=2}}$

$\boxed{\textbf{Lời giải 2 - minhtuyb}}$

- Trừ 2 vế của phương trình $(1)$ cho phương trình $(2)$ ta có:
$$x^2-y^2-4y+4x=0 \Leftrightarrow (x-y)(x+y)+4(x-y)=0 \Leftrightarrow (x-y)(x+y+4)=0$$
*TH 1: Với $x-y=0\Leftrightarrow x=y$, thay $x$ vào phương trình $(1)$ ta có:
$$y^2-4y+4=0\Leftrightarrow (y-2)^2=0\Leftrightarrow y=2\rightarrow x=2$$
*TH 2: Với $x+y+4=0\Leftrightarrow x=-y-4$, thay $x$ vào phương trình $(1)$ ta có:
$$(y+4)^2-4y+4=0 \Leftrightarrow y^2+8y+16-4y+4=0 \Leftrightarrow y^2+4y+20=0$$
$$\Leftrightarrow (y+2)^2+16=0 \text{(phương trình vô nghiệm)}$$
Vậy hệ đã cho có nghiệm duy nhất $x=y=2$

$\boxed{\textbf{Lời giải 3 - binhmetric}}$

Đây là hệ hoán vị vòng quanh nên ta có thể giả sử $x \geq y$.

Khi đó: $0 = {x^2} - 4y + 4 \geq (y-2)^2 \Rightarrow y = 2 \Rightarrow x = 2.$

Vậy hệ có nghiệm duy nhất: $(2; 2)$

$\boxed{\textbf{Lời giải 4 - tieulyly1995}}$

Ta có : $4y-4= x^{2}\geq 0\Leftrightarrow y\geq 1$

Tương tự cũng có $x\geq 1$. Giả sử $x\geq y$
$$\Leftrightarrow 4x-4\geq 4y-4\Leftrightarrow y^{2}\geq x^{2}\Leftrightarrow y\geq x$$
Suy ra $x=y=2 $

$\boxed{\textbf{Bình luận - WWW}}$

Cảm ơn các bạn đã tham gia. Bây giờ chúng ta hãy cũng bình luận về bài toán và những lời giải trên.

Bài toán: Dễ dàng nhận thấy hệ phương trình trên là hệ đối xứng loại II (khi thay đổi vai trò của $x,y$ thì phương trình này trở thành phương trình kia)

Phương pháp thường dùng khi gặp hệ đối xứng đó là trừ vế theo vế như cách làm của bạn minhtuyb. Đây là một cách giải truyền thống.

Lời giải.

- Với cách giải truyền thống trên, chúng ta có thể thấy hiệu quả mà nó mang lại là không nhiều. Mặc dù không sai vì đã đi đúng theo phương pháp nhưng hình thức lại không hay, phải chia trường hợp dài dòng.

- Lập luận của chú binhmetric thiếu chặt chẽ và điều đó đã được đưa ra bởi bạn henry0905.

- Lời giải của tieulyly1995davildark hoàn toàn chính xác. Điều đáng chú ý ở hai lời giải này đó chính là sự sáng tạo, linh hoạt trong hướng suy nghĩ và lập luận. Hai em đã thoát ra khỏi khuôn mẫu của phương pháp. Những đánh giá như thế là rất cần thiết trong việc học và làm Toán.

Lời kết: Qua bài toán này, chúng ta có thể nhận ra một số vấn đề sau:

1. Cần linh hoạt trong hướng suy nghĩ và đưa ra lời giải sáng tạo. Không nên áp dụng máy móc các phương pháp cổ điển, đôi khi những phương pháp đó tỏ ra không hiệu quả. Cần thoát ra khỏi cái cũ để đi đến cái mới.

2. Lập luận chặt chẽ hơn.

Trên đây là một số ý kiến cá nhân của Mr.3W. Mong mọi người góp ý xây dựng.

#8
Crystal

Crystal

    ANGRY BIRDS

  • Hiệp sỹ
  • 5534 Bài viết
Tiếp tục cập nhật ....

#9
minhdat881439

minhdat881439

    Sĩ quan

  • Thành viên
  • 473 Bài viết
Em xin đóng góp 1 bài
BÀI TOÁN: Giải hệ phương trình $\left\{\begin{matrix} \left | xy-18 \right |=12-x^{2} & \\ xy=9+\frac{1}{3}y^{2} & \end{matrix}\right.$
Lời giải 1 - WWW:
Nếu $xy \geqslant 18$, ta có hệ: \[\left\{ \begin{array}{l}
xy - 18 = 12 - {x^2}\\
xy = 9 + \frac{{{y^2}}}{3}
\end{array} \right. \Rightarrow \left\{ \begin{array}{l}
\frac{{{y^2}}}{3} - 9 = 12 - {\left( {\frac{{27 + {y^2}}}{{3y}}} \right)^2}\\
y \ne 0;\,{y^2} \ge 27\\
x = \frac{{27 + {y^2}}}{{3y}}
\end{array} \right. \Rightarrow \left\{\begin{matrix} \begin{bmatrix} y=3\sqrt{3} & \\ x=2\sqrt{3} & \end{bmatrix} & \\ \begin{bmatrix} y=-3\sqrt{3} & \\ x=-2\sqrt{3} & \end{bmatrix} & \end{matrix}\right.\]
Nếu $xy < 18 \Rightarrow {y^2} < 27$, ta có hệ: \[\left\{ \begin{array}{l}
18 - xy = 12 - {x^2}\\
xy = 9 + \frac{{{y^2}}}{3}
\end{array} \right. \Rightarrow \left\{ \begin{array}{l}
9 - \frac{{{y^2}}}{3} = 12 - {\left( {\frac{{27 + {y^2}}}{{3y}}} \right)^2}\\
x = \frac{{27 + {y^2}}}{{3y}}
\end{array} \right. \Rightarrow VN\]
Vậy...

Lời giải 2 - hongcho24031997:
$(1)\Rightarrow x^2\leq 12$

hệ phương trình có nghiệm khi pt (2) có nghiệm:

$\Leftrightarrow y^2-3xy+27=0$ có no

$\Leftrightarrow \Delta= 9x^2-4.27\geq 0\Leftrightarrow x^2\geq 12$

$\Rightarrow x^2= 12$
$\Rightarrow \begin{bmatrix} x=2\sqrt{3}\Rightarrow y=3\sqrt{3} & \\ x=-2\sqrt{3}\Rightarrow y= -3\sqrt{3}& \end{bmatrix}$
Vậy...
Lời giải 3 - hongcho24031997:
$$\left\{\begin{matrix}\left | xy-18 \right |=12-x^2 & \\xy=9 +\frac{1}{3}y^2 & \end{matrix}\right.$$

$$\Leftrightarrow \left\{\begin{matrix}\left | xy-18 \right |+x^2=12 & \\\frac{4}{3}xy=12+\frac{4}{9}y^2 & \end{matrix}\right.$$

$\Rightarrow x^2-\frac{4}{3}xy+\frac{4}{9}y^2+|xy-18|=0$

$\Rightarrow (x-\frac{2}{3}y)^2+|xy-18|=0$

$\Rightarrow \left\{\begin{matrix} x=\frac{2}{3}y & & \\ xy=18 & & \end{matrix}\right.$
$\Leftrightarrow \left\{\begin{matrix} x=\frac{2}{3}y & \\ y^{2}=27 & \end{matrix}\right. \Leftrightarrow \left\{\begin{matrix} \begin{bmatrix} y=3\sqrt{3} & \\ x=2\sqrt{3} & \end{bmatrix} & \\ \begin{bmatrix} y=-3\sqrt{3} & \\ x=-2\sqrt{3} & \end{bmatrix} & \end{matrix}\right.$
Vậy....

----
@WWW: Để gõ dấu hoặc em dùng:
\[\left[ \begin{array}{l} nghiem_1
\\
nghiem_2
\end{array} \right.\]

Đừng ngại học hỏi. Kiến thức là vô bờ, là một kho báu mà ta luôn có thể mang theo dể dàng


Trần Minh Đạt tự hào là thành viên VMF


#10
donghaidhtt

donghaidhtt

    Sĩ quan

  • Thành viên
  • 494 Bài viết
Bài toán:
Giải hệ: $\left\{\begin{matrix} x^{2}-y^{2}=4(1)\\ x^{3}-y^{3}=8(2) \end{matrix}\right.$
Lời giải:
Cách 1:
$(2)\Leftrightarrow x^{3}=8+y^{3}$
$\Leftrightarrow x=\sqrt[3]{8+y^{3}}$
$\Leftrightarrow x^{2}=\sqrt[3]{(8+y^{3})^{2}}(*)$
Thay $(*)$ vào $(1)$ ta có $(1)\Leftrightarrow \sqrt[3]{(8+y^{3})^{2}}-y^{2}=4$
$\Leftrightarrow (8+y^{3})^{2}=(4+y^{2})^{3}$
$\Leftrightarrow 12y^{4}-16y^{3}+48y^{2}=0$
$\Leftrightarrow \begin{bmatrix} y=0\\ 3y^{2}-4y+12=0 \end{bmatrix}$
$\Leftrightarrow y=0$
$\Leftrightarrow x=2$
Loại trường hợp $3y^{2}-4y+12=0$ vì $\Delta ^{'}=-32<0$
Vậy hệ có nghiệm: $(x;y)=(2;0)$
Cách 2:
Nhẩm nghiệm thấy $(x;y)=(2;0)$ nên ta đặt: $y=kx$
Hệ pt được viết lại:
$\left\{\begin{matrix} x^{2}(1-k^{2})=4\\ x^{3}(1-k^{3})=8 \end{matrix}\right.$
Nhận thấy $x=0;y=0$ không phải là nghiệm.
Nhận thấy $k=1$ thì hệ vô nghiệm.
Nên ta xét $x\neq 0;k\neq 1$
Hệ: $\Leftrightarrow \left\{\begin{matrix} (1-k^{2})^{3}=\frac{64}{x^{6}}(3)\\ (1-k^{3})^{2}=\frac{64}{x^{6}}(4) \end{matrix}\right.$
Lấy $(3)-(4):2k^{6}-3k^{4}-2k^{3}+3k^{2}=0$
$\Leftrightarrow \begin{bmatrix} k^{2}=0\\ (k-1)^{2}(2k^{2}+4k+3)=0 \end{bmatrix}$
$\Leftrightarrow k=0$
$\Leftrightarrow y=0$
$y=0 \Leftrightarrow \left\{\begin{matrix} x^{2}=4\\ x^{3}=8 \end{matrix}\right.$
$\Leftrightarrow x=2$
Vậy hệ có nghiệm: $(x;y)=(2;0)$
Cách 3:
$(1)\Leftrightarrow x^{2}=(y+2)^{2}-4y$
Nhận thấy $y=-2$ không thỏa mãn để hệ có nghiệm nên: $(1)\Leftrightarrow x^{2}=(y+2)^{2}-4y$
$\Leftrightarrow \frac{3}{2}x^{2}(y+2)=\frac{3}{2}(y+3)^{3}-6y(y+2)(*)$
$(2)\Leftrightarrow x^{3}=(y+2)^{3}-6y(y+2)(**)$
$(*)-(**):x^{3}-\frac{3}{2}x^{2}(y+2)+\frac{(y+2)^{3}}{2}=0$
$\Leftrightarrow 2x^{3}-3x^{2}(y+2)+(y+2)^{3}=0$
$\Leftrightarrow 2a^{3}-3a^{2}b+b^{3}=0$
$\Leftrightarrow (a-b)^{2}(2a+b)=0$
$\Leftrightarrow \begin{bmatrix} x=y+2\\ 2x=-(y+2) \end{bmatrix}$
Thay lần lượt vào $(1)$: $x=y+2\Leftrightarrow x=2\wedge y=0$
$2x=-(y+2)\Leftrightarrow 3y^{2}-4y+12=0$ vô nghiệm do $\Delta ^{'}=-32< 0$
Vậy hệ có nghiệm: $(x;y)=(2;0)$

Cách 4:
Lời giải của bạn Haruki bên onluyentoan.vn

Mình cũng có một cách! :D
Hệ phương trình đã cho có thể viết lại thành:
$$\left\{\begin{matrix}
(x-y)(x+y)=4 (1)\\(x-y)(x^2 + xy + y^2)=8 (2)

\end{matrix}\right.$$

Ta nhận thấy: $x\neq \pm y$.
Đặt $S= x+y$ và $P=xy$ với $S^2 \geq 4P$. Khi đó lấy (2) chia (1) ta suy ra được: $P=S^2 - 2S$.
Mặt khác:
Từ (2) suy ra được: $x > y$ do $x^2 + xy + y^2 >0$. Nhờ đó từ (1) ta tiếp tục có $x>-y$ .
$\Rightarrow$ $x> |y|$.
Do đó ta có: $(x-y)^{2}=8S-3S^{2}\Leftrightarrow x-y=\sqrt{8S-3S^2}$. Từ đó kết hợp với (1) suy ra được:
$$\sqrt{8S-3S^2}.S=4 \Leftrightarrow (S-2)^2 (3S^2 +4S +4) =0.$$
Vì $3S^2 +4S +4>0$
nên $S=2$ $\Rightarrow$ $P = 0$ $\Rightarrow$ $(x;y)=(2;0)$ là thỏa mãn điều kiện và hệ phương trình.
Vậy nghiệm của hệ phương trình là: $(x;y)=(2;0)$.


Bài viết đã được chỉnh sửa nội dung bởi donghaidhtt: 10-07-2012 - 23:24


#11
khanh3570883

khanh3570883

    Trung úy

  • Thành viên
  • 905 Bài viết
Tiếp tục, một bài toán với nhiều lời giải và nhận xét (đọc ở đâu đó không nhớ!)
Bài toán: Cho a, b, c là các số thực dương thay đổi bất kì. Chứng minh rằng:
\[{\left( {\frac{{b + c}}{a} + \frac{{c + a}}{b} + \frac{{a + b}}{c}} \right)^2} \ge 4\left( {ab + bc + ca} \right)\left( {\frac{1}{{{a^2}}} + \frac{1}{{{b^2}}} + \frac{1}{{{c^2}}}} \right)\]
Lời giải 1: Dễ thấy bất đẳng thức ban đầu tương tương với mỗi bất đẳng thức trong dãy sau:
\[{\left[ {ab\left( {a + b} \right) + bc\left( {b + c} \right) + ca\left( {c + a} \right)} \right]^2} \ge 4\left( {a + b + c} \right)\left( {{a^2}{b^2} + {b^2}{c^2} + {c^2}{a^2}} \right)\]
\[\sum {{a^2}{b^2}{{\left( {a + b} \right)}^2}} + 2abc\left[ {\sum {a\left( {a + b} \right)\left( {a + c} \right)} } \right] \ge 4\left\{ {\sum {{a^3}{b^3}} + abc\left[ {\sum {ab\left( {a + b} \right)} } \right]} \right\}\]
Để ý:
\[\sum {{a^2}{b^2}{{\left( {a + b} \right)}^2}} - 4\left( {\sum {{a^3}{b^3}} } \right) = \sum {{a^2}{b^2}{{\left( {a - b} \right)}^2}} \ge 0\]
và
\[2abc\left[ {\sum {a\left( {a + b} \right)\left( {a + c} \right)} } \right] - 4\left\{ {abc\left[ {\sum {ab\left( {a + b} \right)} } \right]} \right\} = 2abc\left[ {{a^3} + {b^3} + {c^3} + 3abc - \sum {ab\left( {a + b} \right)} } \right] \ge 0\]
do đó bất đẳng thức ban đầu đúng. Vậy ta có đpcm.

Lời giải 2: Giả sử $b = \max \left\{ {a,b,c} \right\}$
Áp dụng bất đẳng thức AM-GM như sau:
\[{\left( {\frac{{b + c}}{a} + \frac{{c + a}}{b} + \frac{{a + b}}{c}} \right)^2} = {\left[ {\left( {\frac{a}{b} + \frac{b}{a} + \frac{a}{c}} \right) + \left( {\frac{b}{c} + \frac{c}{b} + \frac{c}{a}} \right)} \right]^2} \ge 4\left( {\frac{a}{b} + \frac{b}{a} + \frac{a}{c}} \right)\left( {\frac{b}{c} + \frac{c}{b} + \frac{c}{a}} \right)\]
Vậy ta cần chứng minh:
\[\begin{array}{l}
\left( {\frac{a}{b} + \frac{b}{a} + \frac{a}{c}} \right)\left( {\frac{b}{c} + \frac{c}{b} + \frac{c}{a}} \right) \ge \left( {ab + bc + ca} \right)\left( {\frac{1}{{{a^2}}} + \frac{1}{{{b^2}}} + \frac{1}{{{c^2}}}} \right) \\
\Leftrightarrow \frac{{\left( {b - a} \right)\left( {b - c} \right)}}{{ca}} \ge 0 \\
\end{array}\]
Đúng vì $b = \max \left\{ {a,b,c} \right\}$.
Vậy ta có đpcm.

Lời giải 3: Giả sử b nằm giữa a và c.
Áp dụng bất đẳng thức AM-GM như sau:
\[4\left( {ab + bc + ca} \right)\left( {\frac{1}{{{a^2}}} + \frac{1}{{{b^2}}} + \frac{1}{{{c^2}}}} \right) \le {\left[ {\frac{{ab + bc + ca}}{{ca}} + ca\left( {\frac{1}{{{a^2}}} + \frac{1}{{{b^2}}} + \frac{1}{{{c^2}}}} \right)} \right]^2}\]
Như vậy ta cần chứng minh:
\[\begin{array}{l}
\frac{{b + c}}{a} + \frac{{c + a}}{b} + \frac{{a + b}}{c} \ge \frac{{ab + bc + ca}}{{ca}} + ca\left( {\frac{1}{{{a^2}}} + \frac{1}{{{b^2}}} + \frac{1}{{{c^2}}}} \right) \\
\Leftrightarrow \frac{{\left( {a - b} \right)\left( {b - c} \right)}}{{{b^2}}} \ge 0 \\
\end{array}\]
Đúng vì b nằm giữa a và c.
Vậy ta có đpcm.

Nhận xét:
+) Lời giải đầu chỉ là phép biến đổi tương đương.
+) Về 2 lời giải sau:
Nhận thấy bất đẳng thức có dạng ${A^2} \ge 4BC$ với:
\[A = \frac{{b + c}}{a} + \frac{{c + a}}{b} + \frac{{a + b}}{c},B = ab + bc + ca,C = \frac{1}{{{a^2}}} + \frac{1}{{{b^2}}} + \frac{1}{{{c^2}}}\]
Bởi vậy ta liên tưởng đến bất đẳng thức: ${\left( {x + y} \right)^2} \ge 4xy$
Do đó ta có hai hướng giải quyết:
1. Biểu diễn A bằng X + Y với X, Y là một đại lượng thích hợp rồi chứng minh $XY \ge BC$
2. Biểu diễn $BC = \frac{B}{D}CD$ với D là một đại lượng thích hợp rồi chứng minh $A \ge \frac{B}{D} + CD$
Ta cần chọn sao cho bất đẳng thức cần chứng minh sau cùng là đơn giản nhất có thể, tức là nó có càng nhiều nhân tử chung càng tốt.

Hướng 1: Ta viết lại A và BC như sau:
\[\begin{array}{l}
A = \frac{b}{a} + \frac{c}{a} + \frac{c}{b} + \frac{a}{b} + \frac{a}{c} + \frac{b}{c} = X + Y \\
BC = \frac{a}{c} + \frac{c}{b} + \frac{b}{a} + \frac{a}{b} + \frac{b}{c} + \frac{c}{a} + \frac{{ca}}{{{b^2}}} + \frac{{ab}}{{{c^2}}} + \frac{{bc}}{{{a^2}}} \\
\end{array}\]
Để ý trong BC có phần tử $\frac{{ca}}{{{b^2}}}$ nên ta cần có $\frac{a}{b}$ và $\frac{c}{b}$ ở X và Y tương ứng.
\[X = \frac{a}{b} + ...,Y = \frac{c}{b} + ...\]
Mặt khác, trong BC có phần tử $\frac{a}{b}$, mà ở Y đã có $\frac{c}{b}$ nên cần có phần tử $\frac{a}{c}$ ở trong X
\[X = \frac{a}{b} + \frac{a}{c} + ...,Y = \frac{c}{b} + ...\]
Tiếp tục, trong BC có phần tử $\frac{{ab}}{{{c^2}}}$, nên ta cần có $\frac{a}{c}$ và $\frac{b}{c}$ ở X và Y tương ứng
\[X = \frac{a}{b} + \frac{a}{c} + ...,Y = \frac{c}{b} + \frac{b}{c} + ...\]
Tiếp tục ta sẽ chọn được X và Y, chẳng hạn:
\[X = \frac{a}{b} + \frac{b}{a} + \frac{a}{c},Y = \frac{b}{c} + \frac{c}{b} + \frac{c}{a}\]
và ta có lời giải thứ hai. Lưu ý cách chọn X và Y ở trên không phải là duy nhất.

Hướng 2: Xét hiệu sau:
\[A - \frac{B}{D} - CD = \frac{{b + c}}{a} + \frac{{c + a}}{b} + \frac{{a + b}}{c} - \frac{{ab + bc + ca}}{D} - D\left( {\frac{1}{{{a^2}}} + \frac{1}{{{b^2}}} + \frac{1}{{{c^2}}}} \right)\]
Trong hiệu trên hệ số của biến b là:
\[\frac{1}{c} + \frac{1}{a} - \frac{{c + a}}{D}\]
như vậy để thu gọn bất đẳng thức, ta chọn sao cho hệ số của biến b bằng không.
Nếu chọn $D = ca$ thì
\[\begin{array}{l}
A - \frac{B}{D} - CD = \frac{{b + c}}{a} + \frac{{c + a}}{b} + \frac{{a + b}}{c} - \frac{{ab + bc + ca}}{{ca}} - ca\left( {\frac{1}{{{a^2}}} + \frac{1}{{{b^2}}} + \frac{1}{{{c^2}}}} \right) \\
= \frac{{\left( {a - b} \right)\left( {b - c} \right)}}{{{b^2}}} \\
\end{array}\]
và ta có lời giải thứ ba.
+) Cần nhấn mạnh rằng, ẩn chứa trong một lời giải luôn có một kĩ xảo chỉ hướng.

THẬT THÀ THẲNG THẮN THƯỜNG THUA THIỆT

LƯƠN LẸO LUỒN LỎI LẠI LEO LÊN

 

Một ngày nào đó ta sẽ trở lại và lợi hại hơn xưa


#12
Ispectorgadget

Ispectorgadget

    Nothing

  • Quản lý Toán Phổ thông
  • 2946 Bài viết
Bài toán:

Cho $a,b,c>0$ thỏa mãn:$abc=1$.Chứng minh BĐT kép sau:


$$\frac{1}{a+b+1}+\frac{1}{b+c+1}+\frac{1}{c+a+1} \le \frac{1}{a+2}+\frac{1}{b+2}+\frac{1}{c+2} \le 1$$
  • Chứng minh: $$\frac{1}{a+2}+\frac{1}{b+2}+\frac{1}{c+2} \le 1$$
Cách 1: Ispectorgadget
Đặt $x=\frac{1}{a+2};y=\frac{1}{b+2};z=\frac{1}{c+2}\Rightarrow a=\frac{1-2x}{x};b=\frac{1-2y}{y};c=\frac{1-2z}{z}$
Ta có: $abc=1\Rightarrow \frac{(1-2x)(1-2y)(1-2z)}{xyz}=1\Leftrightarrow (1-2x)(1-2y)(1-2z)=xyz$
Do đó ta cần chứng minh $x+y+z\leq 1$
Giả sử ngược lại: $x+y+z>1$
$(1-2x)<(x+y+z)-2z=y+x-z$
$(1-2y)<(x+y+z)-2y=x+z-y$
$(1-2z)<(x+y+z)-2z=x+y-z$
Nhân lại ta có:$xyz=(1-2x)(1-2y)(1-2z)< (x+y-z)(x+z-y)(y+z-x)$
Điều này mâu thuẫn với BĐT Schur $xyz\geq (x+y-z)(x+z-y)(y+z-x)$
Do đó giả sử trên của ta là sai! Do đó BĐT ban đầu là đúng!
Dấu "=" xảy ra khi và chỉ khi $a=b=c=1$
Cách 2:
darktemplar $$\iff \frac{a}{a+2}+\frac{b}{b+2}+\frac{c}{c+2} \ge 1$$
Theo AM-GM:
$$\frac{a}{a+2}=\frac{a}{a+2\sqrt[3]{abc}}=\frac{\sqrt[3]{a^2}}{\sqrt[3]{a^2}+2\sqrt[3]{bc}} \ge \frac{\sqrt[3]{a^2}}{\sqrt[3]{a^2}+\sqrt[3]{b^2}+\sqrt[3]{c^2}}$$
Thiết lập tương tự và cộng lại ta có điều cần chứng minh
Cách 3: Đặt $a;b;c$ lần lượt là $x^2;y^2;z^2$ với với điều kiện $xyz=1$ . Bất đẳng thức trên viết lại thành
$$ \frac{1}{x^2+2} +\frac{1}{y^2+2}+\frac{1}{z^2+2}\le 1$$ Bất đẳng này tương đương với $$\left (\frac{1}{2}-\frac{1}{x^2+2} \right ) +\left (\frac{1}{2}-\frac{1}{y^2+2} \right )+\left (\frac{1}{2}-\frac{1}{z^2+2} \right )\ge \frac{3}{2}-1,$$ hay là $$\frac{x^2}{x^2+2} +\frac{y^2}{y^2+2}+\frac{z^2}{z^2+2}\ge 1.$$ Sử dụng bất đẳng thức Cauchy-Schwarz, ta có $$ \frac{x^2}{x^2+2} +\frac{y^2}{y^2+2}+\frac{z^2}{z^2+2}\ge \frac{(x+y+z)^2}{x^2+y^2+z^2+6}.$$ Cuối cùng ta sẽ chứng minh $$\frac{(x+y+z)^2}{x^2+y^2+z^2+6}\ge 1,$$ Hiển nhiên đúng theo kết quả ở trên. Giá tri lớn nhất của $P$ là $1$ đạt được khi $a=b=c=1.$
  • Chứng minh: $$\frac{1}{a+b+1}+\frac{1}{b+c+1}+\frac{1}{c+a+1} \le \frac{1}{a+2}+\frac{1}{b+2}+\frac{1}{c+2}$$
Cách 1': Đặt $x=a+b+c;y=ab+bc+ac$
BĐT cần chứng minh $$\Leftrightarrow \frac{x^2+4x+y+3}{x^2+2x+y+xy}-1\leq \frac{12+4x+y}{9+4x+2y}-1$$
$$\Leftrightarrow \frac{2x+3-xy}{x^2+2x+y+xy}\leq \frac{3-y}{9+4x+2y}$$
Với BĐT cuối,ta quy đồng và sử dụng các BĐT: $x\geq 3;y\geq 3;x^2\geq 3y$, ta có:
$$\frac{5}{3}x^2y\geq 5x^2;\frac{x^2y}{3}\geq y^2;xy^2\geq 9x;5xy\geq 15x;xy\geq 3y$$
Từ đây ta có đpcm. $\square$
Cách 2': darktemplar
Sử dụng 1 BĐT hiển nhiên sau:
$$\frac{2}{a+2}-\frac{b}{ab+b+1}-\frac{1}{a+b+1}=\frac{a(b-1)^2}{(a+2)(ab+b+1)(a+b+1)} \ge 0$$
Và 1 đẳng thức quen thuộc:
$$\frac{a}{ca+a+1}+\frac{b}{ab+b+1}+\frac{c}{bc+c+1}=1$$
Nên ta có:
$$\sum \frac{1}{a+2}-\sum \frac{1}{a+b+1} \ge 1-\sum \frac{1}{a+2}$$
Như vậy ta chỉ cần chứng minh:
$$\frac{1}{a+2}+\frac{1}{b+2}+\frac{1}{c+2} \le 1$$
BĐT này chúng ta đã chứng minh ở trên,nên ta có đpcm.Đẳng thức xảy ra khi $a=b=c=1$.

Còn 2 cách em không ghi tên :) lời giải trong cuốn Old and new Inequalities

Mấy dấu >, < lúc sửa lại bị lỗi tùm lum hết =.=

Bài viết đã được chỉnh sửa nội dung bởi Ispectorgadget: 11-07-2012 - 10:19

►|| The aim of life is self-development. To realize one's nature perfectly - that is what each of us is here for. ™ ♫


#13
Crystal

Crystal

    ANGRY BIRDS

  • Hiệp sỹ
  • 5534 Bài viết
Nếu khi trích dẫn lời giải của một thành viên thì các bạn nhớ kèm theo nick cho mỗi lời giải nhé.

#14
Crystal

Crystal

    ANGRY BIRDS

  • Hiệp sỹ
  • 5534 Bài viết
BÀI TOÁN. Trích đề thi TSĐH khối A 2010

Giải hệ phương trình: $\left\{ \begin{array}{l} \left( {4{x^2} + 1} \right)x + \left( {y - 3} \right)\sqrt {5 - 2y} = 0\\ 4{x^2} + {y^2} + 2\sqrt {3 - 4x} = 7 \end{array} \right.$


Điều kiện: Do điều kiện như nhau nên áp dụng cho 7 lời giải: $x \leqslant \dfrac{3}{4},y \leqslant \dfrac{5}{2}$

$\boxed{\textbf{Lời giải 1}}$

Phương trình thứ nhất của hệ tương đương với:
$$\left( {4{x^2} + 1} \right)2x = \left( {5 - 2y + 1} \right)\sqrt {5 - 2y} \,\,\,\,\,\,\,\,\,\,\,\,(1)$$
Xét hàm số: $f\left( t \right) = \left( {{t^2} + 1} \right)t \Rightarrow f'\left( t \right) = 3{t^2} + 1 > 0 \Rightarrow f$ tăng trên $\mathbb{R}$.

Khi đó: $$(1) \Leftrightarrow f\left( {2x} \right) = f\left( {\sqrt {5 - 2y} } \right) \Leftrightarrow 2x = \sqrt {5 - 2y} \Leftrightarrow \left\{ \begin{array}{l}
x \ge 0\\
y = \dfrac{{5 - 4{x^2}}}{2}
\end{array} \right.$$
Thay vào phương trình thứ hai của hệ ta được:
$$4{x^2} + {\left( {\dfrac{5}{2} - 2{x^2}} \right)^2} + 2\sqrt {3 - 4x} - 7 = 0\,\,\,\,\,\,\,\,\,\,\,\,\,\,\,\,\,\,(2)$$
Thấy $x = 0,x = \dfrac{3}{4}$ không là nghiệm của $(2)$.
Xét hàm số: $$g\left( x \right) = 4{x^2} + {\left( {\dfrac{5}{2} - 2{x^2}} \right)^2} + 2\sqrt {3 - 4x} - 7,\,\,x \in \left( {0,\dfrac{3}{4}} \right]$$
Ta có: $g'\left( x \right) = 8x - 8x\left( {\dfrac{5}{2} - 2{x^2}} \right) - \dfrac{4}{{\sqrt {3 - 4x} }} = 4x\left( {4{x^2} - 3} \right) - \dfrac{4}{{\sqrt {3 - 4x} }} < 0 \Rightarrow g$ giảm.

Lại có: $g\left( {\dfrac{1}{2}} \right) = 0$ do đó phương trình $(2)$ có nghiệm duy nhất $x = \dfrac{1}{2} \Rightarrow y = 2$

Vậy hệ đã cho có nghiệm là $\boxed{\left( {x,y} \right) = \left( {\dfrac{1}{2},2} \right)}$.

$\boxed{\textbf{Lời giải 2}}$

Phương trình thứ nhất của hệ được viết thành:
$$\left( {4{x^2} + 1} \right)x = \left( {3 - y} \right)\sqrt {5 - 2y} \geqslant 0,\forall y \leqslant \dfrac{5}{2} \Rightarrow x \geqslant 0$$
Đặt $$\left\{ \begin{array}{l}
u = 2x;0 \le u \le \dfrac{3}{2}\\
v = \sqrt {5 - 2y} \ge 0 \Rightarrow y = \dfrac{{5 - {v^2}}}{2}
\end{array} \right.$$
Tha vào phương trình thứ nhất ta có: $$\left( {{u^2} + 1} \right)\dfrac{u}{2} + \left( {\dfrac{{5 - {v^2}}}{2} - 3} \right)v = 0 \Leftrightarrow {u^3} + u - {v^3} - v = 0 \Leftrightarrow {u^3} + u = {v^3} + v\,\,\,\,\,\,\,(1)$$
Xét hàm số: $f\left( t \right) = {t^3} + t \Rightarrow f'\left( t \right) = 3{t^2} + 1 > 0 \Rightarrow f$ tăng trên $\mathbb{R}$.
Từ $$(1) \Rightarrow f\left( u \right) = f\left( v \right) \Leftrightarrow u = v$$
Thay vào phương trình thứ hai: $${u^2} + {\left( {\dfrac{{5 - u}}{2}} \right)^2} + 2\sqrt {3 - 2u} = 7 \Leftrightarrow 8\sqrt {3 - 2u} = - {u^4} + 6{u^2} + 3\,\,\,\,\,\,\,\,\,\,\,\,(2)$$
Xét hàm số: $g\left( u \right) = - {u^4} + 6{u^2} + 3\,;0 \le u \le \dfrac{3}{2}$. Lập bảng biến thiên từ đó suy ra $u=1$ là nghiệm của $(2)$ và đó là nghiệm duy nhất do:
* $h\left( u \right) = 8\sqrt {3 - 2u} $ giảm trên $0 \le u \le \dfrac{3}{2}$

* $g\left( u \right) = - {u^4} + 6{u^2} + 3$ tăng trên $0 \le u \le \dfrac{3}{2}$.

Từ đó suy ra hệ có nghiệm là $\boxed{\left( {x,y} \right) = \left( {\dfrac{1}{2},2} \right)}$.

$\boxed{\textbf{Lời giải 3}}$

Từ phương trình thứ nhất của hệ suy ra:
$$\left( {4{x^2} + 1} \right)x = \left( {3 - y} \right)\sqrt {5 - 2y} \Rightarrow \left\{ \begin{array}{l}
VT = 4{x^3} + x \le \dfrac{{39}}{{16}} \Rightarrow VP = \left( {3 - y} \right)\sqrt {5 - 2y} \le \dfrac{{39}}{{16}} \Rightarrow y \ge 0\\
VP \ge 0 \Rightarrow x \ge 0
\end{array} \right.$$
$$ \Rightarrow \left\{ \begin{array}{l}
0 \le x \le \dfrac{3}{4}\\
0 \le y \le \dfrac{5}{2}
\end{array} \right.$$
Xét hàm số: $$f\left( x \right) = \left( {4{x^2} + 1} \right)x \nearrow \left[ {0,\dfrac{3}{4}} \right],f\left( {\dfrac{1}{2}} \right) = 1,g\left( y \right) = \left( {3 - y} \right)\sqrt {5 - 2y} \searrow \left[ {0,\dfrac{5}{2}} \right],g\left( 2 \right) = 1$$
$$h\left( x \right) = 4{x^2} + 2\sqrt {3 - 4x} \searrow \left[ {0,\dfrac{3}{4}} \right],q\left( y \right) = {y^2} \nearrow \left[ {0,\dfrac{5}{2}} \right]$$
$ \bullet $ Với $0 \le x < \dfrac{1}{2} \Rightarrow g\left( y \right) = f\left( x \right) < f\left( {\dfrac{1}{2}} \right) = g\left( 2 \right) \Rightarrow y > 2 \Rightarrow \left\{ \begin{array}{l}
h\left( x \right) > h\left( {\dfrac{1}{2}} \right) = 3\\
q\left( y \right) > q\left( 2 \right) = 4
\end{array} \right. \Rightarrow V{T_{\left( {PT2} \right)}} > V{P_{\left( {PT2} \right)}}$

$ \bullet $ Với $\dfrac{1}{2} < x \le \dfrac{3}{4} \Rightarrow g\left( y \right) = f\left( x \right) > f\left( {\dfrac{1}{2}} \right) = g\left( 2 \right) \Rightarrow y < 2 \Rightarrow \left\{ \begin{array}{l}
h\left( x \right) < h\left( {\dfrac{1}{2}} \right) = 3\\
q\left( y \right) < q\left( 2 \right) = 4
\end{array} \right. \Rightarrow V{T_{\left( {PT2} \right)}} < V{P_{\left( {PT2} \right)}}$

Với $x = \dfrac{1}{2} \Rightarrow y = 2$

Vậy hệ đã cho có nghiệm là $\boxed{\left( {x,y} \right) = \left( {\dfrac{1}{2},2} \right)}$.

$\boxed{\textbf{Lời giải 4}}$

Phương trình thứ nhất của hệ được viết thành:
$$\left( {4{x^2} + 1} \right)2x = \left( {5 - 2y + 1} \right)\sqrt {5 - 2y} \Leftrightarrow 2x = \sqrt {5 - 2y} $$
$$ \Rightarrow \left\{ \begin{array}{l}
x \ge 0\\
y = \dfrac{{5 - 4{x^2}}}{2}
\end{array} \right. \Leftrightarrow \left\{ \begin{array}{l}
x \ge 0\\
{y^2} = \dfrac{{16{x^4} - 40{x^2} + 25}}{4}
\end{array} \right.$$
Thay vào phương trình thứ hai của hệ:
$$16{x^4} - 24{x^2} + 8\sqrt {3 - 4x} - 3 = 0 \Leftrightarrow \left( {16{x^4} - 1} \right) - \left( {24{x^2} - 6} \right) + \left( {8\sqrt {3 - 4x} - 8} \right) = 0$$
$$ \Leftrightarrow \left( {2x - 1} \right)\left[ {\left( {2x + 1} \right)\left( {4{x^2} - 5} \right) - \dfrac{{16}}{{\sqrt {3 - 4x} + 1}}} \right] = 0 \Leftrightarrow \left( {2x - 1} \right)S = 0 \Leftrightarrow x = \dfrac{1}{2}$$
$$0 \le x \le \dfrac{3}{4} \Rightarrow \left\{ \begin{array}{l}
2x + 1 > 0\\
4{x^2} - 5 < 0
\end{array} \right. \Rightarrow S < 0$$
Từ đó suy ra hệ có nghiệm là $\boxed{\left( {x,y} \right) = \left( {\dfrac{1}{2},2} \right)}$.

$\boxed{\textbf{Lời giải 5}}$

Xét phương trình thứ nhất: $$\left( {4x + 1} \right)x + \left( {y - 3} \right)\sqrt {5 - 2y} = 0$$
Đặt $$\left\{ \begin{array}{l}
u = 2x,u \le \dfrac{3}{2}\\
v = \sqrt {5 - 2y} \ge 0
\end{array} \right. \Rightarrow {v^2} = 5 - 2y \Rightarrow y = \dfrac{{5 - {v^2}}}{2} \Rightarrow y - 3 = - \dfrac{{{v^2} + 1}}{2}$$
$$PT(1) \Leftrightarrow \left( {{u^2} + 1} \right)\dfrac{u}{2} - \left( {\dfrac{{{v^2} + 1}}{2}} \right)v = 0 \Leftrightarrow {u^3} + u - {v^3} - v = 0 \Leftrightarrow \left( {u - v} \right)\left( {{u^2} + uv + {v^2} + 1} \right) = 0$$
Vì $${u^2} + uv + {v^2} + 1 = {\left( {u + \dfrac{v}{2}} \right)^2} + \dfrac{{3{v^2}}}{4} + 1 > 0 \Rightarrow u = v \Rightarrow 2x = \sqrt {5 - 2y} $$
Trở về lời giải 1lời giải 2

$\boxed{\textbf{Lời giải 6}}$

Tương tự các lời giải trên, từ phương trình thứ nhất của hệ, suy ra: $2x = \sqrt {5 - 2y} $

Thay $u=2x$ vào phương trình thứ hai của hệ, ta được:
$${u^2} + {y^2} + 2\sqrt {3 - 2u} = 7 \Leftrightarrow {\left( {y - 1} \right)^2} + 2\sqrt {3 - 2u} = 3$$
Đặt $$v = y - 1 \Rightarrow \left\{ \begin{array}{l}
u = \sqrt {3 - 2v} \\
{v^2} + 2\sqrt {3 - 2u} = 3
\end{array} \right.$$
Đặt $$w = \sqrt {3 - 2u} \Rightarrow \left\{ \begin{array}{l}
{w^2} + 2u = 3\\
{u^2} + 2v = 3\\
{v^2} + 2w = 3
\end{array} \right.$$
Từ hệ trên dễ dàng chứng minh được $u = v = w = 1 \Rightarrow x = \dfrac{1}{2},y = 2$

$\boxed{\textbf{Lời giải 7}}$

Đặt $u = \sqrt {5 - 2y} ,v = \sqrt {3 - 4x} ;u,v \ge 0$, khi đó ta có hệ phương trình sau:
$$\left\{ \begin{array}{l}
4{x^3} + x + \left( {\dfrac{{5 - {u^2}}}{2} - 3} \right) = 0\\
4{x^2} + {y^2} + 2v = 7\\
x = \dfrac{{3 - {v^2}}}{4}\\
y = \dfrac{{5 - {u^2}}}{2}
\end{array} \right. \Rightarrow 8{x^3} + 2x - {u^3} - u = \left( {2x - u} \right)\left( {3{x^2} + {{\left( {x + u} \right)}^2}} \right) = 0$$
Suy ra: $$u = 2x \Rightarrow y = \dfrac{{5 - 4{{\left( {\dfrac{{3 - {v^2}}}{4}} \right)}^2}}}{2} \Rightarrow 4{\left( {\dfrac{{3 - {v^2}}}{4}} \right)^2} + {\left( {\dfrac{{5 - 4{{\left( {\dfrac{{3 - {v^2}}}{4}} \right)}^2}}}{2}} \right)^2} + 2v = 7$$
$$ \Leftrightarrow {v^8} - 12{v^6} + 30{v^4} + 36{v^2} + 128v - 183 = 0$$
$$ \Leftrightarrow \left( {v - 1} \right)\left( {{v^7} + {v^6} - 11{v^5} - 11{v^4} + 19{v^3} + 19{v^2} + 55v + 183} \right) = 0$$
$$v \in \left[ {0,\sqrt 3 } \right] \Rightarrow {v^7} + {v^6} - 11{v^5} - 11{v^4} + 19{v^3} + 19{v^2} + 55v + 183 = 0\,\,\,\,vn$$
$$ \Rightarrow v = 1 \Rightarrow \sqrt {3 - 4x} = 1 \Leftrightarrow x = \dfrac{1}{2} \Rightarrow y = 2$$
Từ đó suy ra hệ có nghiệm là $\boxed{\left( {x,y} \right) = \left( {\dfrac{1}{2},2} \right)}$.

#15
donghaidhtt

donghaidhtt

    Sĩ quan

  • Thành viên
  • 494 Bài viết
Bài toán: Giải hệ:
\[\left\{\begin{matrix}
x+y-\sqrt{xy}=3\\
\sqrt{x+1}+\sqrt{y+1}=4
\end{matrix}\right.\]


Lời giải:
Điều kiện: \[\left\{\begin{matrix}
x\geq -1\\
y\geq -1\\
xy\geq 0
\end{matrix}\right.\]
Nhận thấy: \[(1)\Leftrightarrow x+y=3+\sqrt{xy}\geq 3\]
và kết hợp điều kiện ta có \[x,y\geq 0\]
Cách 1:
Đặt $S=x+y\geq 0$ và $P=xy\geq 0$
\[(2)\Leftrightarrow x+y+2+2\sqrt{xy+(x+y)+1}=16\Leftrightarrow 2\sqrt{P+S+1}=14-S\Leftrightarrow \left\{\begin{matrix}
4(P+S+1)=S^{2}-28S+196\\
0\leq
S\leq 14\end{matrix}\right.(*)\]
\[(1)\Leftrightarrow \sqrt{P}=S-3\Leftrightarrow \left\{\begin{matrix}
P=S^{2}-6S+9\\
S\geq
3\end{matrix}\right.(**)\]
Thay $(**)$ vào $(*)$: \[3S^{2}+8S-156=0\Leftrightarrow \begin{bmatrix}
S=6\\
S=\frac{-26}{3}
\end{bmatrix}\Leftrightarrow S=6\]
Thỏa mãn điều kiện \[3\leq S\leq 14\]
Từ đó $P=9$ nên $x=y=3$
Cách 2:
AM-GM: \[x+y\geq 2\sqrt{xy}\Leftrightarrow 3+\sqrt{xy}\geq 2\sqrt{xy}\Leftrightarrow \sqrt{xy}\leq 3(*)\]
CS: \[16=(\sqrt{x+1}+\sqrt{y+1})^{2}\leq (x+y+2).2=(5+\sqrt{xy}).2\Leftrightarrow \sqrt{xy}\geq 3(**)\]
Từ $(*)$ và $(**)$ ta có $\sqrt{xy}=3$
Từ đó được $x+y=6$ và $xy=9$ nên $x=y=3$
Cách 3:
AM-GM: $(2)\Leftrightarrow 4=\frac{\sqrt{(x+1)4}}{2}+\frac{\sqrt{(y+1)4}}{2}\leq \frac{x+y+10}{4}\Rightarrow x+y\geq 6(*)$

AM-GM: $\sqrt{xy}\leq \frac{x+y}{2}\Leftrightarrow 3=x+y-\sqrt{xy}\geq (x+y)-\frac{x+y}{2}\Leftrightarrow x+y\leq 6(**)$
Từ $(*)$ và $(**)$ có $x+y=6$ nên $xy=9$ Từ đó $x=y=3$
Cách 4-5:
\[(2)\Leftrightarrow \sqrt{x+1}-2+\sqrt{y+1}-2=0\Leftrightarrow \frac{x-3}{\sqrt{x+1}+2}+\frac{y-3}{\sqrt{y+1}+2}=0\]
Từ $(1)$ có: \[x-3=\sqrt{xy}-y=\sqrt{y}(\sqrt{x}-\sqrt{y})\]
và \[y-3=\sqrt{xy}-x=\sqrt{x}(\sqrt{y}-\sqrt{x})\]
Nên thay vào có:
\[\begin{bmatrix}
\sqrt{x}=\sqrt{y} \\
\dfrac{\sqrt{y}}{\sqrt{x+1}+2}=\dfrac{\sqrt{x}}{\sqrt{y+1}+2}
\end{bmatrix}\]
+ $\sqrt{x}=\sqrt{y}$ dễ có $x=y=3$
+ \[\dfrac{\sqrt{y}}{\sqrt{x+1}+2}=\dfrac{\sqrt{x}}{\sqrt{y+1}+2}\](*)
Xét $(*)$ có 2 cách:
Cách 4:
$\dfrac{\sqrt{y}}{\sqrt{x+1}+2}=\dfrac{\sqrt{x}}{\sqrt{y+1}+2}$

$\Leftrightarrow
2(\sqrt{x}-\sqrt{y})+\sqrt{x}\sqrt{x+1}-\sqrt{y}\sqrt{y+1}=0$

$\Leftrightarrow 2(\sqrt{x}-\sqrt{y})+\dfrac{x^{2}+x-y^{2}-y}{\sqrt{x}\sqrt{x+1}+\sqrt{y}\sqrt{y+1}}=0$
$\Leftrightarrow \begin{bmatrix}
\sqrt{x}=\sqrt{y}\\
2+\dfrac{(\sqrt{x}+\sqrt{y})(x+y+1)}{\sqrt{x}\sqrt{x+1}+\sqrt{y}\sqrt{y+1}}=0\end{bmatrix}$
$\Leftrightarrow \sqrt{x}=\sqrt{y}$
Cách 5:
+Nếu $x>y>0$
\[\left\{\begin{matrix}
\dfrac{\sqrt{y}}{\sqrt{x+1}+2}<\dfrac{\sqrt{y}}{\sqrt{y+1}+2} \\
\dfrac{\sqrt{x}}{\sqrt{y+1}+2}>\dfrac{\sqrt{y}}{\sqrt{y+1}+2}\\
\dfrac{\sqrt{y}}{\sqrt{x+1}+2}=\dfrac{\sqrt{x}}{\sqrt{y+1}+2}
\end{matrix}\right.\]
Vô nghiệm
+ Nếu $0<x<y$
\[\left\{\begin{matrix}
\dfrac{\sqrt{y}}{\sqrt{x+1}+2}>\dfrac{\sqrt{x}}{\sqrt{x+1}+2} \\
\dfrac{\sqrt{x}}{\sqrt{y+1}+2}<\dfrac{\sqrt{x}}{\sqrt{x+1}+2}\\
\dfrac{\sqrt{y}}{\sqrt{x+1}+2}=\dfrac{\sqrt{x}}{\sqrt{y+1}+2}
\end{matrix}\right.\]
Vô nghiệm
+ Nếu $x=y$ thỏa
Vậy $x=y=3$
Cách 6:
Đặt: \[\left\{\begin{matrix}
\sqrt{x+1}=m+2\geq 0\\
\sqrt{y+1}=n+2\geq 0
\end{matrix}\right.\]
Từ pt $(2)$ ta có $m+n=0$
Và \[\left\{\begin{matrix}
x=m^{2}+4m+3\\
y=n^{2}+4n+3
\end{matrix}\right.\]
chú ý $m+n=0$:
\[(1)\Leftrightarrow m^{2}+n^{2}+4(m+n)+6-\sqrt{(m^{2}+4m+3)(n^{2}+4n+3)}=3\Leftrightarrow \left\{\begin{matrix}
mn\leq \dfrac{3}{2}\\\
3m^{2}n^{2}=22mn
\end{matrix}\right.\Leftrightarrow mn=0\]
nên $m=n=0$ từ đó $x=y=3$
Cách 7:
Đặt: \[\left\{\begin{matrix}
\sqrt{x+1}=m\geq 0\\
\sqrt{y+1}=n\geq 0
\end{matrix}\right.\]

Từ đó
\[\left\{\begin{matrix}
x=m^{2}-1\\
y=n^{2}-1
\end{matrix}\right.\]
Cách làm tương tự, với $m+n=4$ thay vào pt $(1)$
Có được mn=4
Nên $x=y=2$
Cách 8:
Lê Mậu Úy
$(1).2-(2).4\Leftrightarrow
2x+2y-2\sqrt{xy}-6-4\sqrt{x+1}-4\sqrt{y+1}+16=0$
$\Leftrightarrow (\sqrt{x}+\sqrt{y})^{2}+(\sqrt{x+1}-2)^{2}+(\sqrt{y+1}-2)^{2}=0$
$\Leftrightarrow \left\{\begin{matrix}
\sqrt{x}=\sqrt{y}\\
\sqrt{x+1}=2\\
\sqrt{y+1}=2
\end{matrix}\right.$
$\Leftrightarrow x=y=3$
Cách 9:
Đặt $y=kx$
\[(1)\Leftrightarrow (k+1)x-x\sqrt{k}=3\]
\[(2)\Leftrightarrow \sqrt{x+1}+\sqrt{kx+1}=4\Leftrightarrow x(k+1)+2+2\sqrt{(x+1)(k+1)}=16\]
Thay \[(k+1)x=x\sqrt{k}+3\] vào:
$3+x\sqrt{k}+2+2\sqrt{kx^{2}+4+x\sqrt{k}}=16$
$\Leftrightarrow \left\{\begin{matrix}
11\geq x\sqrt{k}\geq 0\\
3(x\sqrt{k})^{2}+26x\sqrt{k}-105=0
\end{matrix}\right.$
$\Leftrightarrow x\sqrt{k}=3$
$\Leftrightarrow k=\frac{9}{x^{2}}$
$\Leftrightarrow xy=9$
$\Leftrightarrow x+y=6\Rightarrow x=y=3$
P/S: Bài này chắc còn có các cách khác nữa, tạm thời chừng này đã Hình đã gửi
Cách 1-2-3 em nhớ có thấy trên diễn đàn rồi nhưng không nhớ tên, đành đánh máy lại. Mấy cách kia tối ni hi sinh giấc ngủ để làm và đánh Latex. Hình đã gửi

Trong đó cách 8 là cách hay và ngắn nhất của bạn em Hình đã gửi

Bài viết đã được chỉnh sửa nội dung bởi donghaidhtt: 16-07-2012 - 03:01


#16
Tham Lang

Tham Lang

    Thượng úy

  • Thành viên
  • 1149 Bài viết

Tiếp tục là một bài toán khá hay, mà theo mình sẽ có rất nhiều cách giải. Mình post đề trước đã, mọi người thử làm rồi mình sẽ post lời giải của mình sau :
Bài toán
Cho $a,b,c$ là các số thực không âm thoả mãn $ab+bc+ca \ne 0$. Chứng minh rằng :
$$\left (ab+\dfrac{c}{a+b}\right )\left (bc+\dfrac{a}{b+c}\right )\left (ca+\dfrac{b}{c+a}\right )\le \dfrac{1}{4}$$

Tập huấn đội tuyển 2009 -Phạm Kim Hùng

2 cách không mấy dễ chịu cho lắm.
Cách 1. [ Võ Quốc Bá Cẩn]
Không mất tính tổng quát, giả sử $a=\max {a,b,c}$ Khi đó, ta có biến đổi sau :
$$\left (ca+\dfrac{b}{c+a}\right )\left (ab+\dfrac{c}{a+b}\right )-a^2bc+\dfrac{c^2a}{a+b}+\dfrac{b^2a}{c+a}+\dfrac{bc}{(a+b)(a+c)}$$
$$=a^2bc+b^2+c^2-\dfrac{bc^2}{a+b}-\dfrac{b^2c}{a+c}+\dfrac{bc}{(a+b)(a+c)}$$
$$(b+c)^2+bc\left [a^2+2-\dfrac{c}{a+b}-\dfrac{b}{c+a}+\dfrac{(a+b+c)^2}{(a+b)(a+c)}\right ]$$
$$=(b+c)^2+bc\left [a^2-1+\dfrac{bc}{(a+b)(a+c)}\right ]$$
Đặt $A=a^2-1+\dfrac{bc}{(a+b)(a+c)}$ (dễ thầy $A\le 0$). Bất đẳng thức cần chứng mình có thể viết lại dưới dạng :
$$\left [(b+c)^2+Abc\right ]\left (bc+\dfrac{a}{b+c}\right )\le \dfrac{1}{4}\Leftrightarrow a(b+c)+Ab^2c^2+bc\left [\dfrac{Aa}{b+c}+(b+c)^2\right ]\le \dfrac{1}{4}$$
Ta có $\dfrac{1}{4}-a(b+c)=\dfrac{(b+c-a)^2}{4}\ge 2bc(b+c-a)^2$ và $Ab^2c^2\le 0$ nên bất đẳng thức này được suy ra từ :
$$bc\left [\dfrac{Aa}{b+c}+(b+c)^2\right ]\le 2bc(b+c-a)^2\Leftrightarrow 2(b+c-a)^2\ge \dfrac{Aa}{b+c}+(b+c)^2$$
Đặt $t=\dfrac{b+c}{2}\le \dfrac{1}{3}$ thì dễ thấy $\dfrac{bc}{(a+b)(a+c)}\le \dfrac{t^2}{(a+t)^2}$ nên chỉ cần chứng minh BĐT mạnh hơn là :
$$2(2t-a)^2\ge \dfrac{a}{2t}\left [a^2-1+\dfrac{t^2}{(a+t)^2}\right ]+4t^2$$
Thay $a=1-2t$ vào. bất đẳng thức trở thành :
$$2\left (16t^2-111t+2\right )\ge \dfrac{1(1-2t)}{2(1-t)^2}$$
Ta có $4(1-t)^2 \ge 4\left (1-\dfrac{1}{3}\right )^2=\dfrac{16}{9}$ và $16t^2-11t+2-t(1-2t)=2(1-3t)^2\ge 0$ nên Bất đẳng thức cuối cùng hiển nhiên đúng. Đẳng thức xảy ra khi $(a,b,c)$ là hoán vị của $\left (\dfrac{1}{2}, \dfrac{1}{2}, 0\right )$.
Cách 2. [ Tham Lang ]
Đặt $\left\{\begin{array}{1}ab+bc+ca=q\\ abc=r \end{array}\right.$ thì sau một sô bước biến đổi siêng năng, ta cần chứng minh :
$$r^2-4r+\dfrac{q^2}{q-r}\le \dfrac{1}{4}$$
Xét hàm số
$$f®=r^2-4r+\dfrac{q^2}{q-r}$$
$$f'®=2r-4+\dfrac{q^2}{(q-r)^2}$$
Ta sẽ chứng minh $f'®\le 0 \Leftrightarrow 2r+\dfrac{q^2}{(q-r)^2}\le 4$.
Lị có $r\le \dfrac{1}{27}$
Nên chỉ cần chứng minh :
$$\dfrac{q^2}{(q-r)^2} \le \dfrac{106}{27}\Leftrightarrow \left (\sqrt{106}-\sqrt{27}\right )q\ge \sqrt{106}r$$
Đúng vì $q\ge 9r$.
Như vậy, hàm nghịch biến. Mặt khác, theo Schur, ta lại có :
$$r\ge \max \left (0, \dfrac{4q-1}{9}\right )$$
Tới đây, ta sẽ xét 2 TH.
  • TH1. $0\ge 4q-1\Leftrightarrow q\le \dfrac{1}{4}$
Lúc này :
$$f®\le f(0) =q \le \dfrac{1}{4}$$
  • TH2. $\dfrac{1}{4}\le q\le \dfrac{1}{3}$.
Ta có :
$$f®\le f\left (\dfrac{4q-1}{9}\right )=\dfrac{(4q-1)^2}{81}-\dfrac{4(4q-1)}{9}+\dfrac{q^2}{1-\dfrac{4q-1}{9}}=\dfrac{80q^3-15q^2+23q+37}{405q+81}$$
Ta cần chứng minh :
$$f\left (\dfrac{4q-1}{9}\right )\le \dfrac{1}{4}$$
$$\dfrac{80q^3-15q^2+23q+37}{405q+81}\le \dfrac{1}{4}$$
$$\Leftrightarrow 320q^3-60q^2-313q+67\le 0$$
$$\Leftrightarrow \left (80q^2+5q-67\right )(4q-1)\le 0$$
Đúng vì $4q-1\ge 0; 80q^2+5q\le \dfrac{80}{9}+\dfrac{5}{3}<67$
Bất đẳng thức đã được chứng minh. Dấu "=" xảy ra khi $(a,b,c)$ là hoán vị của $\left (\dfrac{1}{2}, \dfrac{1}{2}, 0\right )$

Off vĩnh viễn ! Không ngày trở lại.......


#17
Ispectorgadget

Ispectorgadget

    Nothing

  • Quản lý Toán Phổ thông
  • 2946 Bài viết
Bài toán: Cho $a,b,c\geq 0$ thoã mãn: $a^2+b^2+c^2=1$
chứng minh rằng: $$\frac{1}{1-a}+\frac{1}{1-b}+\frac{1}{1-c}\geq \frac{3\sqrt{3}+9}{2}$$
Cách 1: dark templar
Từ giả thuyết,ta suy ra được:
$$\frac{1}{1-a}=\frac{a+1}{b^2+c^2};\frac{1}{1-b}=\frac{b+1}{a^2+c^2};\frac{1}{1-c}=\frac{c+1}{a^2+b^2}$$
Như vậy ta viết lại BĐT dưới dạng sau:
$$\left(\frac{1}{a^2+b^2}+\frac{1}{b^2+c^2}+\frac{1}{c^2+a^2} \right)+\left(\frac{a}{1-a^2}+\frac{b}{1-b^2}+\frac{c}{1-c^2} \right) \ge \frac{3\sqrt{3}+9}{2}$$
Với ngoặc đầu,theo Cauchy-Schwarz:
$$\frac{1}{a^2+b^2}+\frac{1}{b^2+c^2}+\frac{1}{c^2+a^2} \ge \frac{9}{2(a^2+b^2+c^2)}=\frac{9}{2}$$
Với ngoặc sau,ta dễ dàng chứng minh được 1 BĐT luôn đúng sau:
$$\frac{a}{1-a^2} \ge \frac{3\sqrt{3}a^2}{2} \iff 3\sqrt{3}a^3+2 \ge 3\sqrt{3}a$$
(Luôn đúng theo AM-GM)
Vậy:$$\frac{a}{1-a^2}+\frac{b}{1-b^2}+\frac{c}{1-c^2} \ge \frac{3\sqrt{3}(a^2+b^2+c^2)}{2}=\frac{3\sqrt{3}}{2}$$
Suy ra đpcm.Đẳng thức xảy ra khi $a=b=c=\frac{1}{\sqrt{3}}$.
Cách 2: Ispectorgadget
Lời giải:
Vì $a,b,c$ dương và $a^2+b^2+c^2=1$ nên suy ra $a,b,c\in (0;1)$. Xét hàm số
$$h(t)=\frac{1}{1-t}-\frac{9+6\sqrt{3}}{4}t^2,\forall t\in (0;1)$$
$$h'(t)=\frac{1}{(1-t)^2}-\frac{9+6\sqrt{3}}{2}t,\forall t \in (0;1)$$
$$h'(t)=0\Leftrightarrow \frac{1}{(1-t)^2}=\frac{9+6\sqrt{3}}{2}t\Leftrightarrow 3\sqrt{3}t^3-6\sqrt{3}t^2+3\sqrt{3}t-42\sqrt{3}=0$$
$$\Leftrightarrow t_1=\frac{6\sqrt{3}-3-\sqrt{36\sqrt{3}-27}}{6\sqrt{3}}\in (0;1)$$
$$t_3=\frac{6\sqrt{3}-3+\sqrt{36\sqrt{3}-27}}{6\sqrt{3}}>1$$
Kẻ bảng biến thiên ta có
$$h(t)\geq \frac{3}{4},\forall t\in (0;1)$$
$$\frac{1}{1-t}\geq \frac{9+6\sqrt{3}}{4}t^2+\frac{3}{4},t\in (0;1)$$
Thay t lần lượt bởi $a,b,c$ rồi cộng các vế cùng chiều ta được
$$\frac{1}{1-a}+\frac{1}{1-b}+\frac{1}{1-c}\geq \frac{9+6\sqrt{3}}{4}(a^2+b^2+c^2)+\frac{9}{4}$$
$$\Leftrightarrow \frac{1}{1-a}+\frac{1}{1-b}+\frac{1}{1-c}\geq \frac{9+3\sqrt{3}}{2}\blacksquare$$

►|| The aim of life is self-development. To realize one's nature perfectly - that is what each of us is here for. ™ ♫


#18
Crystal

Crystal

    ANGRY BIRDS

  • Hiệp sỹ
  • 5534 Bài viết
BÀI TOÁN. Chứng minh rằng với mọi số thực dương $x,y,z$ thỏa mãn $x(x+y+z)=3yz$, ta có:
$$(x+y)^{3}+(x+z)^{3}+3(x+y)(x+z)(y+z)\leq 5(y+z)^{3}$$
GIẢI.

Cách 1 [HÀ QUỐC ĐẠT]

Đặt $a=x+y;b=x+z;c=y+z \Rightarrow x=\dfrac{a+b-c}{2};y=\dfrac{a+c-b}{2};z=\dfrac{b+c-a}{2}$
Khi đó: $$x(x+y+z)=3yz = \dfrac{a+b-c}{2}.\dfrac{a+b+c}{2}=3\dfrac{a+c-b}{2}.\dfrac{b+c-a}{2}$$
$$\Leftrightarrow (a+b)^{2}-c^{2}=3[c^{2}-(b-a)^{2}]$$
$$\Leftrightarrow a^{2}+b^{2}-c^{2}+2ab=-3a^{2}-3b^{2}+3c^{2}+6ab$$
$$\Leftrightarrow 4c^{2}=4a^{2}+4b^{2}-4ab$$
$$\Leftrightarrow c^{2}=a^{2}+b^{2}-ab\geq \dfrac{(a+b)^{2}}{4}\geq ab \Leftrightarrow 2c\geq a+b$$
Ta có: $3c^{3}\geq 3abc$
$$\Leftrightarrow 2c^{3}-(a^{3}+b^{3})=2c^{3}-(a+b)(a^{2}-ab+b^{2})=2c^{3}-(a+b)c^{2}=c^{2}[2c-(a+b)]\geq 0$$
$$\Leftrightarrow 2c^{3}\geq a^{3}+b^{3} \Leftrightarrow 5c^{3}\geq a^{3}+b^{3}+3abc$$
$$\Leftrightarrow 5(y+z)^{3}\geq (x+y)^{3}+(y+z)^{3}+3(x+y)(y+z)(z+x)$$
Dẫu "=" xảy ra khi $x=y=z=1$

Cách 2 [WWW]
Đặt: $y = ax;z = bx$. Khi đó $(1)$ trở thành: $x\left( {x + ax + bx} \right) = 3ab{x^2} \Leftrightarrow 1 + a + b = 3ab\,\,\,(3)$

và $(2)$ trở thành: ${\left( {x + ax} \right)^3} + {\left( {x + bx} \right)^3} + 3\left( {x + ax} \right)\left( {ax + bx} \right)\left( {bx + x} \right) \le 5{\left( {ax + bx} \right)^3}$
$$ \Leftrightarrow {\left( {1 + a} \right)^3} + {\left( {1 + b} \right)^3} + 3\left( {1 + a} \right)\left( {1 + b} \right)\left( {a + b} \right) \le 5{\left( {a + b} \right)^3}\,\,\,(4)$$
Vì $(3)$ và $(4)$ là những biểu thức đối xứng với $a, b$ nên đặt $S = a + b;\,P = ab$, khi đó
$$\left\{ \begin{array}{l}{S^2} \ge 4P\\1 + S = 3P\end{array} \right. \Leftrightarrow \left\{ \begin{array}{l}P = \dfrac{{1 + S}}{3}\\3{S^2} - 4S - 4 \ge 0\end{array} \right. \Leftrightarrow \left\{ \begin{array}{l}P = \dfrac{{1 + S}}{3}\\S \ge 2\end{array} \right.$$
Suy ra $$\left( {1 + a} \right)\left( {1 + b} \right) = 1 + a + b + ab = 1 + S + \dfrac{{1 + S}}{3} = \dfrac{{4\left( {1 + S} \right)}}{3}$$
$${\left( {1 + a} \right)^3} + {\left( {1 + b} \right)^3} = {\left( {2 + a + b} \right)^3} - 3\left( {1 + a} \right)\left( {1 + b} \right)\left( {2 + a + b} \right) = {\left( {2 + S} \right)^3} - 4\left( {1 + S} \right)\left( {2 + S} \right)$$
$$\left( 4 \right) \Leftrightarrow {\left( {2 + S} \right)^3} - 4\left( {{S^2} + 3S + 2} \right) + 4S\left( {1 + S} \right) \le 5{S^3}$$
$$ \Leftrightarrow 2{S^2} - 3S - 2 \ge 0 \Leftrightarrow \left( {2S + 1} \right)\left( {S - 2} \right) \ge 0\,\,\,\text{ luôn đúng do}\,\,S \ge 2$$
Vậy ta có đpcm.

Cách 3: [WWW]
$\left( 1 \right) \Leftrightarrow {x^2} + xy + x{\rm{z}} = 3yz \Rightarrow {(x + y)^2} + {(x + z)^2} = 2{(y + z)^2} - {(y - z)^2}$

$ \Rightarrow {\left( {\dfrac{{x + y}}{{y + z}}} \right)^2} + {\left( {\dfrac{{x + z}}{{y + z}}} \right)^2} = 2 - {\left( {\dfrac{{x + y}}{{y + z}} - \dfrac{{x + z}}{{y + z}}} \right)^2}\,\,\,\,\left( 3 \right)$

Đặt $u = \dfrac{{x + y}}{{y + z}},\,\,v = \dfrac{{x + z}}{{y + z}}$. Từ (3) $ \Rightarrow {u^2} + {v^2} = 2 - {(u - v)^2} \Rightarrow {u^2} + {v^2} - uv = 1\,\,\,\,\,\left( 4 \right)$

Khi đó ta có: BĐT $ \Leftrightarrow {\left( {\dfrac{{x + y}}{{y + z}}} \right)^3} + {\left( {\dfrac{{x + z}}{{y + z}}} \right)^3} + 3\left( {\dfrac{{x + y}}{{y + z}}} \right)\left( {\dfrac{{x + z}}{{y + z}}} \right) \le 5 \Leftrightarrow {u^3} + {v^3} + 3uv \le 5$

$ \Leftrightarrow (u + v)({u^2} - uv + {v^2}) + 3uv \le 5 \Leftrightarrow u + v + 3uv \le 5\,\,\,\left( 5 \right)$

Mặt khác từ (1) ta có: $uv = 1 - {(u - v)^2} \le 1\,\,\,\left( 6 \right)$

và ${(u + v)^2} = 1 + 3uv \le 1 + \dfrac{3}{4}{(u + v)^2} \Rightarrow {(u + v)^2} \le 4 \Rightarrow u + v \le 2\,\,\,\left( 7 \right)$

Từ $(6)$ và $(7)$ ta có đpcm.




1 người đang xem chủ đề

0 thành viên, 1 khách, 0 thành viên ẩn danh